Xem bài viết đơn
Old 04-01-2014, 08:33 AM   #10
namdung
Administrator

 
Tham gia ngày: Feb 2009
Đến từ: Tp Hồ Chí Minh
Bài gởi: 1,343
Thanks: 209
Thanked 4,066 Times in 778 Posts
Gửi tin nhắn qua Yahoo chát tới namdung
Thực ra ta có thể chứng minh kết quả mạnh hơn như sau:

Nếu $P(x) = (x^2-7x+6)^{2n} + 13 $ có thể phân tích thành tích của 2 đa thức Q(x), S(x) với hệ số nguyên thì Q(x) và S(x) đều có bậc 2n.

Từ đây với n > 1 bài toán trở nên hiển nhiên, còn với n = 1 ta có thể kiểm tra trực tiếp rằng $(x^2-7x+6)^2 + 13 $ bất khả quy.

Thật vậy, giả sử P(x) = Q(x).S(x).

Gọi $x_1, x_2, ..., x_{4n} $ là các nghiệm phức của P(x) thì sẽ là tích của các thừa số $(x-x_i) $. Đánh số lại nếu cần, ta giả sử $Q(x) = (x-x_1)(x-x_2)...(x-x_k) $ với $1 \le k < 4n $.

Ta có $((x_i-1)(x_i-6))^{2n} = -13 $. Từ đây suy ra $|(x_i-1)(x_i-6)| = 13^{\frac{1}{2n}} $. (*)

Mặt khác $(1-x_1)...(1-x_k) = Q(1) $ nguyên nên $|(1-x_1)...(1-x_k)| $ nguyên. Tương tự $|(6-x_1)...(6-x_k)| $ nguyên.

Từ đây suy ra $m = |(x_1-1)(x_1-6)(x_2-1)(x_2-6)...(x_k-1)(x_k-6)| $ nguyên.

Nhưng theo (*) thì $m = 13^{k/2n} $

Suy ra k = 2n. Vậy Q(x), S(x) đều phải có bậc là 2n.

Tôi có cảm nhận rằng đa thức P(x) là bất khả quy, tuy nhiên cần thời gian để kiểm tra chắc chắn.

Lời giải trên đây dựa trên ý tưởng của 2 bài toán kinh điển sau:

1. Tìm tất cả giá trị n sao cho đa thức $x^n + 4 $ có thể phân tích thành tích của hai đa thức khác hằng số với hệ số nguyên.

2. (IMO 1993) Chứng minh rằng với mọi n> 1, đa thức $x^n - 5x^{n-1} + 3 $ bất khả quy.
[RIGHT][I][B]Nguồn: MathScope.ORG[/B][/I][/RIGHT]
 

thay đổi nội dung bởi: namdung, 04-01-2014 lúc 08:35 AM
namdung is offline   Trả Lời Với Trích Dẫn
The Following 8 Users Say Thank You to namdung For This Useful Post:
12121993 (04-01-2014), DaiToan (04-01-2014), DogLover (05-01-2014), hakudoshi (04-01-2014), huynhcongbang (04-01-2014), luugiangnam (05-01-2014), quangvinht2 (04-01-2014), thaygiaocht (04-01-2014)
 
[page compression: 10.03 k/11.13 k (9.90%)]